Application of Chebyschev inequality

The name of the pictureThe name of the pictureThe name of the pictureClash Royale CLAN TAG#URR8PPP











up vote
4
down vote

favorite
2












enter image description here



I want to prove the inequality above. On the extreme ends we get a clear application of AM-GM, and I want to use the chebyschev inequality for the middle but was having trouble.



My Attempt:



Since Chebyschevs inequalty is
enter image description here



We can square the left hand side of our inequality with the term to its right to get



$$fraca+b+c3cdot fraca+b+c3geq fracca+b^2+ca3$$



My problem



I would've wanted to get $dfracab+bc+ca3$ on the right instead. Did I apply the inequality wrong, or does it follow that $dfracab+bc+ca3$ is less than or equal to $dfraca+b+c3cdot dfraca+b+c3$?







share|cite|improve this question

























    up vote
    4
    down vote

    favorite
    2












    enter image description here



    I want to prove the inequality above. On the extreme ends we get a clear application of AM-GM, and I want to use the chebyschev inequality for the middle but was having trouble.



    My Attempt:



    Since Chebyschevs inequalty is
    enter image description here



    We can square the left hand side of our inequality with the term to its right to get



    $$fraca+b+c3cdot fraca+b+c3geq fracca+b^2+ca3$$



    My problem



    I would've wanted to get $dfracab+bc+ca3$ on the right instead. Did I apply the inequality wrong, or does it follow that $dfracab+bc+ca3$ is less than or equal to $dfraca+b+c3cdot dfraca+b+c3$?







    share|cite|improve this question























      up vote
      4
      down vote

      favorite
      2









      up vote
      4
      down vote

      favorite
      2






      2





      enter image description here



      I want to prove the inequality above. On the extreme ends we get a clear application of AM-GM, and I want to use the chebyschev inequality for the middle but was having trouble.



      My Attempt:



      Since Chebyschevs inequalty is
      enter image description here



      We can square the left hand side of our inequality with the term to its right to get



      $$fraca+b+c3cdot fraca+b+c3geq fracca+b^2+ca3$$



      My problem



      I would've wanted to get $dfracab+bc+ca3$ on the right instead. Did I apply the inequality wrong, or does it follow that $dfracab+bc+ca3$ is less than or equal to $dfraca+b+c3cdot dfraca+b+c3$?







      share|cite|improve this question













      enter image description here



      I want to prove the inequality above. On the extreme ends we get a clear application of AM-GM, and I want to use the chebyschev inequality for the middle but was having trouble.



      My Attempt:



      Since Chebyschevs inequalty is
      enter image description here



      We can square the left hand side of our inequality with the term to its right to get



      $$fraca+b+c3cdot fraca+b+c3geq fracca+b^2+ca3$$



      My problem



      I would've wanted to get $dfracab+bc+ca3$ on the right instead. Did I apply the inequality wrong, or does it follow that $dfracab+bc+ca3$ is less than or equal to $dfraca+b+c3cdot dfraca+b+c3$?









      share|cite|improve this question












      share|cite|improve this question




      share|cite|improve this question








      edited Jul 31 at 18:57









      Michael Rozenberg

      87.5k1577179




      87.5k1577179









      asked Jul 31 at 14:30









      john fowles

      1,088817




      1,088817




















          2 Answers
          2






          active

          oldest

          votes

















          up vote
          2
          down vote



          accepted










          Firstly, your inequality is wrong.



          Try $a=1$, $b=-1$ and $c=0$.



          For non-negatives $a$, $b$ and $c$ we see that $(a,b,c)$ and $(a,b,c)$ they are the same ordered.



          This, by Chebyshov we obtain:
          $$3(acdot a+bcdot b+ccdot c)geq(a+b+c)(a+b+c)$$or
          $$3(a^2+b^2+c^2)geq(a+b+c)^2$$ or
          $$3(a^2+b^2+c^2+2(ab+ac+bc))geq(a+b+c)^2+2(ab+ac+bc)$$ or
          $$(a+b+c)^2geq3(ab+ac+bc)$$ or
          $$fraca+b+c3geqsqrtfracab+ac+bc3.$$
          By the same way we obtain:
          $$3(abcdot ab+accdot ac+bccdot bc)geq (ab+ac+bc)(ab+ac+bc)$$ or
          $$3(a^2b^2+a^2c^2+b^2c^2)geq(ab+ac+bc)^2$$ or
          $$(ab+ac+bc)^2geq3abc(a+b+c).$$
          Now, $(ab,ac,bc)$ and $(c,b,a)$ they are opposite ordered.



          Thus, by Chebyshov again we obtain:
          $$(ab+ac+bc)^3geq3abc(a+b+c)(ab+ac+bc)=3abc(c+b+a)(ab+ac+bc)geq$$
          $$geq3abccdot3(ccdot ab+bcdot ac+acdot bc)=27a^2b^2c^2,$$
          which gives
          $$sqrtfracab+ac+bc3geqsqrt[3]abc.$$






          share|cite|improve this answer




























            up vote
            1
            down vote













            As you've noted here the use of the Chebyshev's Inequality doesn't help much. However we can make use of the AM-GM inequality.



            Squaring both sides and expanding them we get that the inequality is equivalent to: $(a+b+c)^2 ge 3(ab+bc+ca)$, which is equivalent to $a^2 + b^2 + c^2 ge ab + bc + ca$.



            The last equality is true, as we have that $a^2 + b^2 ge 2ab$, $b^2 + c^2 ge 2bc$ and $c^2 + a^2 ge 2ac$. Add the inequalities to get the final answer.






            share|cite|improve this answer





















              Your Answer




              StackExchange.ifUsing("editor", function ()
              return StackExchange.using("mathjaxEditing", function ()
              StackExchange.MarkdownEditor.creationCallbacks.add(function (editor, postfix)
              StackExchange.mathjaxEditing.prepareWmdForMathJax(editor, postfix, [["$", "$"], ["\\(","\\)"]]);
              );
              );
              , "mathjax-editing");

              StackExchange.ready(function()
              var channelOptions =
              tags: "".split(" "),
              id: "69"
              ;
              initTagRenderer("".split(" "), "".split(" "), channelOptions);

              StackExchange.using("externalEditor", function()
              // Have to fire editor after snippets, if snippets enabled
              if (StackExchange.settings.snippets.snippetsEnabled)
              StackExchange.using("snippets", function()
              createEditor();
              );

              else
              createEditor();

              );

              function createEditor()
              StackExchange.prepareEditor(
              heartbeatType: 'answer',
              convertImagesToLinks: true,
              noModals: false,
              showLowRepImageUploadWarning: true,
              reputationToPostImages: 10,
              bindNavPrevention: true,
              postfix: "",
              noCode: true, onDemand: true,
              discardSelector: ".discard-answer"
              ,immediatelyShowMarkdownHelp:true
              );



              );








               

              draft saved


              draft discarded


















              StackExchange.ready(
              function ()
              StackExchange.openid.initPostLogin('.new-post-login', 'https%3a%2f%2fmath.stackexchange.com%2fquestions%2f2868100%2fapplication-of-chebyschev-inequality%23new-answer', 'question_page');

              );

              Post as a guest






























              2 Answers
              2






              active

              oldest

              votes








              2 Answers
              2






              active

              oldest

              votes









              active

              oldest

              votes






              active

              oldest

              votes








              up vote
              2
              down vote



              accepted










              Firstly, your inequality is wrong.



              Try $a=1$, $b=-1$ and $c=0$.



              For non-negatives $a$, $b$ and $c$ we see that $(a,b,c)$ and $(a,b,c)$ they are the same ordered.



              This, by Chebyshov we obtain:
              $$3(acdot a+bcdot b+ccdot c)geq(a+b+c)(a+b+c)$$or
              $$3(a^2+b^2+c^2)geq(a+b+c)^2$$ or
              $$3(a^2+b^2+c^2+2(ab+ac+bc))geq(a+b+c)^2+2(ab+ac+bc)$$ or
              $$(a+b+c)^2geq3(ab+ac+bc)$$ or
              $$fraca+b+c3geqsqrtfracab+ac+bc3.$$
              By the same way we obtain:
              $$3(abcdot ab+accdot ac+bccdot bc)geq (ab+ac+bc)(ab+ac+bc)$$ or
              $$3(a^2b^2+a^2c^2+b^2c^2)geq(ab+ac+bc)^2$$ or
              $$(ab+ac+bc)^2geq3abc(a+b+c).$$
              Now, $(ab,ac,bc)$ and $(c,b,a)$ they are opposite ordered.



              Thus, by Chebyshov again we obtain:
              $$(ab+ac+bc)^3geq3abc(a+b+c)(ab+ac+bc)=3abc(c+b+a)(ab+ac+bc)geq$$
              $$geq3abccdot3(ccdot ab+bcdot ac+acdot bc)=27a^2b^2c^2,$$
              which gives
              $$sqrtfracab+ac+bc3geqsqrt[3]abc.$$






              share|cite|improve this answer

























                up vote
                2
                down vote



                accepted










                Firstly, your inequality is wrong.



                Try $a=1$, $b=-1$ and $c=0$.



                For non-negatives $a$, $b$ and $c$ we see that $(a,b,c)$ and $(a,b,c)$ they are the same ordered.



                This, by Chebyshov we obtain:
                $$3(acdot a+bcdot b+ccdot c)geq(a+b+c)(a+b+c)$$or
                $$3(a^2+b^2+c^2)geq(a+b+c)^2$$ or
                $$3(a^2+b^2+c^2+2(ab+ac+bc))geq(a+b+c)^2+2(ab+ac+bc)$$ or
                $$(a+b+c)^2geq3(ab+ac+bc)$$ or
                $$fraca+b+c3geqsqrtfracab+ac+bc3.$$
                By the same way we obtain:
                $$3(abcdot ab+accdot ac+bccdot bc)geq (ab+ac+bc)(ab+ac+bc)$$ or
                $$3(a^2b^2+a^2c^2+b^2c^2)geq(ab+ac+bc)^2$$ or
                $$(ab+ac+bc)^2geq3abc(a+b+c).$$
                Now, $(ab,ac,bc)$ and $(c,b,a)$ they are opposite ordered.



                Thus, by Chebyshov again we obtain:
                $$(ab+ac+bc)^3geq3abc(a+b+c)(ab+ac+bc)=3abc(c+b+a)(ab+ac+bc)geq$$
                $$geq3abccdot3(ccdot ab+bcdot ac+acdot bc)=27a^2b^2c^2,$$
                which gives
                $$sqrtfracab+ac+bc3geqsqrt[3]abc.$$






                share|cite|improve this answer























                  up vote
                  2
                  down vote



                  accepted







                  up vote
                  2
                  down vote



                  accepted






                  Firstly, your inequality is wrong.



                  Try $a=1$, $b=-1$ and $c=0$.



                  For non-negatives $a$, $b$ and $c$ we see that $(a,b,c)$ and $(a,b,c)$ they are the same ordered.



                  This, by Chebyshov we obtain:
                  $$3(acdot a+bcdot b+ccdot c)geq(a+b+c)(a+b+c)$$or
                  $$3(a^2+b^2+c^2)geq(a+b+c)^2$$ or
                  $$3(a^2+b^2+c^2+2(ab+ac+bc))geq(a+b+c)^2+2(ab+ac+bc)$$ or
                  $$(a+b+c)^2geq3(ab+ac+bc)$$ or
                  $$fraca+b+c3geqsqrtfracab+ac+bc3.$$
                  By the same way we obtain:
                  $$3(abcdot ab+accdot ac+bccdot bc)geq (ab+ac+bc)(ab+ac+bc)$$ or
                  $$3(a^2b^2+a^2c^2+b^2c^2)geq(ab+ac+bc)^2$$ or
                  $$(ab+ac+bc)^2geq3abc(a+b+c).$$
                  Now, $(ab,ac,bc)$ and $(c,b,a)$ they are opposite ordered.



                  Thus, by Chebyshov again we obtain:
                  $$(ab+ac+bc)^3geq3abc(a+b+c)(ab+ac+bc)=3abc(c+b+a)(ab+ac+bc)geq$$
                  $$geq3abccdot3(ccdot ab+bcdot ac+acdot bc)=27a^2b^2c^2,$$
                  which gives
                  $$sqrtfracab+ac+bc3geqsqrt[3]abc.$$






                  share|cite|improve this answer













                  Firstly, your inequality is wrong.



                  Try $a=1$, $b=-1$ and $c=0$.



                  For non-negatives $a$, $b$ and $c$ we see that $(a,b,c)$ and $(a,b,c)$ they are the same ordered.



                  This, by Chebyshov we obtain:
                  $$3(acdot a+bcdot b+ccdot c)geq(a+b+c)(a+b+c)$$or
                  $$3(a^2+b^2+c^2)geq(a+b+c)^2$$ or
                  $$3(a^2+b^2+c^2+2(ab+ac+bc))geq(a+b+c)^2+2(ab+ac+bc)$$ or
                  $$(a+b+c)^2geq3(ab+ac+bc)$$ or
                  $$fraca+b+c3geqsqrtfracab+ac+bc3.$$
                  By the same way we obtain:
                  $$3(abcdot ab+accdot ac+bccdot bc)geq (ab+ac+bc)(ab+ac+bc)$$ or
                  $$3(a^2b^2+a^2c^2+b^2c^2)geq(ab+ac+bc)^2$$ or
                  $$(ab+ac+bc)^2geq3abc(a+b+c).$$
                  Now, $(ab,ac,bc)$ and $(c,b,a)$ they are opposite ordered.



                  Thus, by Chebyshov again we obtain:
                  $$(ab+ac+bc)^3geq3abc(a+b+c)(ab+ac+bc)=3abc(c+b+a)(ab+ac+bc)geq$$
                  $$geq3abccdot3(ccdot ab+bcdot ac+acdot bc)=27a^2b^2c^2,$$
                  which gives
                  $$sqrtfracab+ac+bc3geqsqrt[3]abc.$$







                  share|cite|improve this answer













                  share|cite|improve this answer



                  share|cite|improve this answer











                  answered Jul 31 at 18:55









                  Michael Rozenberg

                  87.5k1577179




                  87.5k1577179




















                      up vote
                      1
                      down vote













                      As you've noted here the use of the Chebyshev's Inequality doesn't help much. However we can make use of the AM-GM inequality.



                      Squaring both sides and expanding them we get that the inequality is equivalent to: $(a+b+c)^2 ge 3(ab+bc+ca)$, which is equivalent to $a^2 + b^2 + c^2 ge ab + bc + ca$.



                      The last equality is true, as we have that $a^2 + b^2 ge 2ab$, $b^2 + c^2 ge 2bc$ and $c^2 + a^2 ge 2ac$. Add the inequalities to get the final answer.






                      share|cite|improve this answer

























                        up vote
                        1
                        down vote













                        As you've noted here the use of the Chebyshev's Inequality doesn't help much. However we can make use of the AM-GM inequality.



                        Squaring both sides and expanding them we get that the inequality is equivalent to: $(a+b+c)^2 ge 3(ab+bc+ca)$, which is equivalent to $a^2 + b^2 + c^2 ge ab + bc + ca$.



                        The last equality is true, as we have that $a^2 + b^2 ge 2ab$, $b^2 + c^2 ge 2bc$ and $c^2 + a^2 ge 2ac$. Add the inequalities to get the final answer.






                        share|cite|improve this answer























                          up vote
                          1
                          down vote










                          up vote
                          1
                          down vote









                          As you've noted here the use of the Chebyshev's Inequality doesn't help much. However we can make use of the AM-GM inequality.



                          Squaring both sides and expanding them we get that the inequality is equivalent to: $(a+b+c)^2 ge 3(ab+bc+ca)$, which is equivalent to $a^2 + b^2 + c^2 ge ab + bc + ca$.



                          The last equality is true, as we have that $a^2 + b^2 ge 2ab$, $b^2 + c^2 ge 2bc$ and $c^2 + a^2 ge 2ac$. Add the inequalities to get the final answer.






                          share|cite|improve this answer













                          As you've noted here the use of the Chebyshev's Inequality doesn't help much. However we can make use of the AM-GM inequality.



                          Squaring both sides and expanding them we get that the inequality is equivalent to: $(a+b+c)^2 ge 3(ab+bc+ca)$, which is equivalent to $a^2 + b^2 + c^2 ge ab + bc + ca$.



                          The last equality is true, as we have that $a^2 + b^2 ge 2ab$, $b^2 + c^2 ge 2bc$ and $c^2 + a^2 ge 2ac$. Add the inequalities to get the final answer.







                          share|cite|improve this answer













                          share|cite|improve this answer



                          share|cite|improve this answer











                          answered Jul 31 at 14:47









                          Stefan4024

                          27.8k52974




                          27.8k52974






















                               

                              draft saved


                              draft discarded


























                               


                              draft saved


                              draft discarded














                              StackExchange.ready(
                              function ()
                              StackExchange.openid.initPostLogin('.new-post-login', 'https%3a%2f%2fmath.stackexchange.com%2fquestions%2f2868100%2fapplication-of-chebyschev-inequality%23new-answer', 'question_page');

                              );

                              Post as a guest













































































                              Comments

                              Popular posts from this blog

                              Relationship between determinant of matrix and determinant of adjoint?

                              Color the edges and diagonals of a regular polygon

                              What is the equation of a 3D cone with generalised tilt?